XOR-free sets: Maximum density?












4












$begingroup$


It is known that sum-free
subsets of $mathbb{N}$ can have
natural density at most
$frac{1}{2}$. This density is achieved by the odd numbers: the sum of two
odd numbers is even.



I ask now a similar question for XOR rather than addition.
For $a,b in mathbb{N}$, define $a oplus b$ as follows:
Represent $a ge b$ as binary numbers, pad the smaller $b$ with zeros,
and take the bit-wise XOR of the binary representation.
For example, $35 oplus 15 = 44$:
begin{eqnarray}
35 = & ;100011\
15 = & ;001111\
oplus = & ;101100
end{eqnarray}

The condition for a set $S subset mathbb{N}$ to be XOR-free is that, for any $a,b in S$, $a oplus b notin S$.




Q. What is the largest density of an XOR-free subset $S$ of $mathbb{N}$?




Again the odd numbers with density $frac{1}{2}$ are XOR-free.
I am not seeing an argument that $frac{1}{2}$ is the maximum possible density.










share|cite|improve this question









$endgroup$












  • $begingroup$
    Maybe a parity argument helps? As a guide, consider all numbers with an odd number of 1-bits. Gerhard "Are Parity Arguments Also Even?" Paseman, 2019.02.24.
    $endgroup$
    – Gerhard Paseman
    2 hours ago






  • 1




    $begingroup$
    You can get close. For discussion sake, focus on the last bit. You can't have two pairs of numbers which differ only in the last bit as both pairs sum to 1. This means out of 2^n bit patterns, at most n + 2^(n-1) will avoid having two pairs sum to a single bit. Gerhard "At Least We're Asymptotically Right" Paseman, 2019.02.24.
    $endgroup$
    – Gerhard Paseman
    1 hour ago












  • $begingroup$
    Related question (bounded numbers in {0,1,..,2^n-1}): mathoverflow.net/questions/293198/…
    $endgroup$
    – kodlu
    1 hour ago


















4












$begingroup$


It is known that sum-free
subsets of $mathbb{N}$ can have
natural density at most
$frac{1}{2}$. This density is achieved by the odd numbers: the sum of two
odd numbers is even.



I ask now a similar question for XOR rather than addition.
For $a,b in mathbb{N}$, define $a oplus b$ as follows:
Represent $a ge b$ as binary numbers, pad the smaller $b$ with zeros,
and take the bit-wise XOR of the binary representation.
For example, $35 oplus 15 = 44$:
begin{eqnarray}
35 = & ;100011\
15 = & ;001111\
oplus = & ;101100
end{eqnarray}

The condition for a set $S subset mathbb{N}$ to be XOR-free is that, for any $a,b in S$, $a oplus b notin S$.




Q. What is the largest density of an XOR-free subset $S$ of $mathbb{N}$?




Again the odd numbers with density $frac{1}{2}$ are XOR-free.
I am not seeing an argument that $frac{1}{2}$ is the maximum possible density.










share|cite|improve this question









$endgroup$












  • $begingroup$
    Maybe a parity argument helps? As a guide, consider all numbers with an odd number of 1-bits. Gerhard "Are Parity Arguments Also Even?" Paseman, 2019.02.24.
    $endgroup$
    – Gerhard Paseman
    2 hours ago






  • 1




    $begingroup$
    You can get close. For discussion sake, focus on the last bit. You can't have two pairs of numbers which differ only in the last bit as both pairs sum to 1. This means out of 2^n bit patterns, at most n + 2^(n-1) will avoid having two pairs sum to a single bit. Gerhard "At Least We're Asymptotically Right" Paseman, 2019.02.24.
    $endgroup$
    – Gerhard Paseman
    1 hour ago












  • $begingroup$
    Related question (bounded numbers in {0,1,..,2^n-1}): mathoverflow.net/questions/293198/…
    $endgroup$
    – kodlu
    1 hour ago
















4












4








4





$begingroup$


It is known that sum-free
subsets of $mathbb{N}$ can have
natural density at most
$frac{1}{2}$. This density is achieved by the odd numbers: the sum of two
odd numbers is even.



I ask now a similar question for XOR rather than addition.
For $a,b in mathbb{N}$, define $a oplus b$ as follows:
Represent $a ge b$ as binary numbers, pad the smaller $b$ with zeros,
and take the bit-wise XOR of the binary representation.
For example, $35 oplus 15 = 44$:
begin{eqnarray}
35 = & ;100011\
15 = & ;001111\
oplus = & ;101100
end{eqnarray}

The condition for a set $S subset mathbb{N}$ to be XOR-free is that, for any $a,b in S$, $a oplus b notin S$.




Q. What is the largest density of an XOR-free subset $S$ of $mathbb{N}$?




Again the odd numbers with density $frac{1}{2}$ are XOR-free.
I am not seeing an argument that $frac{1}{2}$ is the maximum possible density.










share|cite|improve this question









$endgroup$




It is known that sum-free
subsets of $mathbb{N}$ can have
natural density at most
$frac{1}{2}$. This density is achieved by the odd numbers: the sum of two
odd numbers is even.



I ask now a similar question for XOR rather than addition.
For $a,b in mathbb{N}$, define $a oplus b$ as follows:
Represent $a ge b$ as binary numbers, pad the smaller $b$ with zeros,
and take the bit-wise XOR of the binary representation.
For example, $35 oplus 15 = 44$:
begin{eqnarray}
35 = & ;100011\
15 = & ;001111\
oplus = & ;101100
end{eqnarray}

The condition for a set $S subset mathbb{N}$ to be XOR-free is that, for any $a,b in S$, $a oplus b notin S$.




Q. What is the largest density of an XOR-free subset $S$ of $mathbb{N}$?




Again the odd numbers with density $frac{1}{2}$ are XOR-free.
I am not seeing an argument that $frac{1}{2}$ is the maximum possible density.







nt.number-theory integer-sequences






share|cite|improve this question













share|cite|improve this question











share|cite|improve this question




share|cite|improve this question










asked 2 hours ago









Joseph O'RourkeJoseph O'Rourke

85.3k16233700




85.3k16233700












  • $begingroup$
    Maybe a parity argument helps? As a guide, consider all numbers with an odd number of 1-bits. Gerhard "Are Parity Arguments Also Even?" Paseman, 2019.02.24.
    $endgroup$
    – Gerhard Paseman
    2 hours ago






  • 1




    $begingroup$
    You can get close. For discussion sake, focus on the last bit. You can't have two pairs of numbers which differ only in the last bit as both pairs sum to 1. This means out of 2^n bit patterns, at most n + 2^(n-1) will avoid having two pairs sum to a single bit. Gerhard "At Least We're Asymptotically Right" Paseman, 2019.02.24.
    $endgroup$
    – Gerhard Paseman
    1 hour ago












  • $begingroup$
    Related question (bounded numbers in {0,1,..,2^n-1}): mathoverflow.net/questions/293198/…
    $endgroup$
    – kodlu
    1 hour ago




















  • $begingroup$
    Maybe a parity argument helps? As a guide, consider all numbers with an odd number of 1-bits. Gerhard "Are Parity Arguments Also Even?" Paseman, 2019.02.24.
    $endgroup$
    – Gerhard Paseman
    2 hours ago






  • 1




    $begingroup$
    You can get close. For discussion sake, focus on the last bit. You can't have two pairs of numbers which differ only in the last bit as both pairs sum to 1. This means out of 2^n bit patterns, at most n + 2^(n-1) will avoid having two pairs sum to a single bit. Gerhard "At Least We're Asymptotically Right" Paseman, 2019.02.24.
    $endgroup$
    – Gerhard Paseman
    1 hour ago












  • $begingroup$
    Related question (bounded numbers in {0,1,..,2^n-1}): mathoverflow.net/questions/293198/…
    $endgroup$
    – kodlu
    1 hour ago


















$begingroup$
Maybe a parity argument helps? As a guide, consider all numbers with an odd number of 1-bits. Gerhard "Are Parity Arguments Also Even?" Paseman, 2019.02.24.
$endgroup$
– Gerhard Paseman
2 hours ago




$begingroup$
Maybe a parity argument helps? As a guide, consider all numbers with an odd number of 1-bits. Gerhard "Are Parity Arguments Also Even?" Paseman, 2019.02.24.
$endgroup$
– Gerhard Paseman
2 hours ago




1




1




$begingroup$
You can get close. For discussion sake, focus on the last bit. You can't have two pairs of numbers which differ only in the last bit as both pairs sum to 1. This means out of 2^n bit patterns, at most n + 2^(n-1) will avoid having two pairs sum to a single bit. Gerhard "At Least We're Asymptotically Right" Paseman, 2019.02.24.
$endgroup$
– Gerhard Paseman
1 hour ago






$begingroup$
You can get close. For discussion sake, focus on the last bit. You can't have two pairs of numbers which differ only in the last bit as both pairs sum to 1. This means out of 2^n bit patterns, at most n + 2^(n-1) will avoid having two pairs sum to a single bit. Gerhard "At Least We're Asymptotically Right" Paseman, 2019.02.24.
$endgroup$
– Gerhard Paseman
1 hour ago














$begingroup$
Related question (bounded numbers in {0,1,..,2^n-1}): mathoverflow.net/questions/293198/…
$endgroup$
– kodlu
1 hour ago






$begingroup$
Related question (bounded numbers in {0,1,..,2^n-1}): mathoverflow.net/questions/293198/…
$endgroup$
– kodlu
1 hour ago












1 Answer
1






active

oldest

votes


















4












$begingroup$

Let $ain S$ be some fixed element. Note that $aoplus b le a + b$. Let $N$ be some big number. Put $M = [1, ldots , N]cap S$. We have $aoplus M cap M = varnothing$. We also have $aoplus M subset [1, ldots , N + a]$. Therefore $2|M| le N + a$ or $|M| le frac{N}{2} + frac{a}{2}$. Since $a$ is fixed taking limit $Nto infty$ yields the desired result.



UPD Here is an answer for a perhaps more interesting question: what is limsup of the biggest density of the subset of $[0, ldots , N]$ which is XOR-free. Note that if $N = 2^k - 1$ for some $kin mathbb{N}$ then $|S| le 2^{k-1}$. Indeed, for any $a, bin [0, ldots, N]$ we have $aoplus b in [0, ldots , N]$ . Since $|Soplus S| ge |S|$ and $Scap (Soplus S) = varnothing$ we get $2|S| le (N+1)$ or $|S| le 2^{k-1}$.



For the general case assume that $2^k le N < 2^{k+1} - 1$. Then on the one hand $|S| le 2^k$ since $Ssubset [0, ldots , 2^{k+1}-1]$ and on the other hand $|S| le 2^{k-1} + (N - 2^k) = N - 2^{k-1}$ since $|Scap [0, ldots , 2^k - 1]| le 2^{k-1}$. Therefore $3|S| le 2N$ or $|S| le frac{2N}{3}$.



Here is an example (found partly via computer search) which shows that density $frac{2}{3}$ is possible: put $N = 6*2^k$ and let $S$ be a set of all numbers consisting of $k + 3$ digits such that first three of them is from the following set $M = {(0, 1, 0), (0, 1, 1), (1, 0, 0), (1, 0, 1)}$. Note that $Soplus S cap S = varnothing$, all elements of $S$ are not greater than $N$ and $|S| = 4*2^k$. Therefore $frac{|S|}{N} = frac{2}{3}$.






share|cite|improve this answer











$endgroup$









  • 2




    $begingroup$
    The example can be described more simply as the integers whose fist two digits are $(0,1)$ or $(1,0)$, that is, $S = [2^{k+1}, 3 cdot 2^{k+1})$. Those digits sum to $1$, so in any element of $S oplus S$ they sum to $0$, whence $S$ is disjoint from $S oplus S$.
    $endgroup$
    – Noam D. Elkies
    47 mins ago











Your Answer





StackExchange.ifUsing("editor", function () {
return StackExchange.using("mathjaxEditing", function () {
StackExchange.MarkdownEditor.creationCallbacks.add(function (editor, postfix) {
StackExchange.mathjaxEditing.prepareWmdForMathJax(editor, postfix, [["$", "$"], ["\\(","\\)"]]);
});
});
}, "mathjax-editing");

StackExchange.ready(function() {
var channelOptions = {
tags: "".split(" "),
id: "504"
};
initTagRenderer("".split(" "), "".split(" "), channelOptions);

StackExchange.using("externalEditor", function() {
// Have to fire editor after snippets, if snippets enabled
if (StackExchange.settings.snippets.snippetsEnabled) {
StackExchange.using("snippets", function() {
createEditor();
});
}
else {
createEditor();
}
});

function createEditor() {
StackExchange.prepareEditor({
heartbeatType: 'answer',
autoActivateHeartbeat: false,
convertImagesToLinks: true,
noModals: true,
showLowRepImageUploadWarning: true,
reputationToPostImages: 10,
bindNavPrevention: true,
postfix: "",
imageUploader: {
brandingHtml: "Powered by u003ca class="icon-imgur-white" href="https://imgur.com/"u003eu003c/au003e",
contentPolicyHtml: "User contributions licensed under u003ca href="https://creativecommons.org/licenses/by-sa/3.0/"u003ecc by-sa 3.0 with attribution requiredu003c/au003e u003ca href="https://stackoverflow.com/legal/content-policy"u003e(content policy)u003c/au003e",
allowUrls: true
},
noCode: true, onDemand: true,
discardSelector: ".discard-answer"
,immediatelyShowMarkdownHelp:true
});


}
});














draft saved

draft discarded


















StackExchange.ready(
function () {
StackExchange.openid.initPostLogin('.new-post-login', 'https%3a%2f%2fmathoverflow.net%2fquestions%2f324044%2fxor-free-sets-maximum-density%23new-answer', 'question_page');
}
);

Post as a guest















Required, but never shown

























1 Answer
1






active

oldest

votes








1 Answer
1






active

oldest

votes









active

oldest

votes






active

oldest

votes









4












$begingroup$

Let $ain S$ be some fixed element. Note that $aoplus b le a + b$. Let $N$ be some big number. Put $M = [1, ldots , N]cap S$. We have $aoplus M cap M = varnothing$. We also have $aoplus M subset [1, ldots , N + a]$. Therefore $2|M| le N + a$ or $|M| le frac{N}{2} + frac{a}{2}$. Since $a$ is fixed taking limit $Nto infty$ yields the desired result.



UPD Here is an answer for a perhaps more interesting question: what is limsup of the biggest density of the subset of $[0, ldots , N]$ which is XOR-free. Note that if $N = 2^k - 1$ for some $kin mathbb{N}$ then $|S| le 2^{k-1}$. Indeed, for any $a, bin [0, ldots, N]$ we have $aoplus b in [0, ldots , N]$ . Since $|Soplus S| ge |S|$ and $Scap (Soplus S) = varnothing$ we get $2|S| le (N+1)$ or $|S| le 2^{k-1}$.



For the general case assume that $2^k le N < 2^{k+1} - 1$. Then on the one hand $|S| le 2^k$ since $Ssubset [0, ldots , 2^{k+1}-1]$ and on the other hand $|S| le 2^{k-1} + (N - 2^k) = N - 2^{k-1}$ since $|Scap [0, ldots , 2^k - 1]| le 2^{k-1}$. Therefore $3|S| le 2N$ or $|S| le frac{2N}{3}$.



Here is an example (found partly via computer search) which shows that density $frac{2}{3}$ is possible: put $N = 6*2^k$ and let $S$ be a set of all numbers consisting of $k + 3$ digits such that first three of them is from the following set $M = {(0, 1, 0), (0, 1, 1), (1, 0, 0), (1, 0, 1)}$. Note that $Soplus S cap S = varnothing$, all elements of $S$ are not greater than $N$ and $|S| = 4*2^k$. Therefore $frac{|S|}{N} = frac{2}{3}$.






share|cite|improve this answer











$endgroup$









  • 2




    $begingroup$
    The example can be described more simply as the integers whose fist two digits are $(0,1)$ or $(1,0)$, that is, $S = [2^{k+1}, 3 cdot 2^{k+1})$. Those digits sum to $1$, so in any element of $S oplus S$ they sum to $0$, whence $S$ is disjoint from $S oplus S$.
    $endgroup$
    – Noam D. Elkies
    47 mins ago
















4












$begingroup$

Let $ain S$ be some fixed element. Note that $aoplus b le a + b$. Let $N$ be some big number. Put $M = [1, ldots , N]cap S$. We have $aoplus M cap M = varnothing$. We also have $aoplus M subset [1, ldots , N + a]$. Therefore $2|M| le N + a$ or $|M| le frac{N}{2} + frac{a}{2}$. Since $a$ is fixed taking limit $Nto infty$ yields the desired result.



UPD Here is an answer for a perhaps more interesting question: what is limsup of the biggest density of the subset of $[0, ldots , N]$ which is XOR-free. Note that if $N = 2^k - 1$ for some $kin mathbb{N}$ then $|S| le 2^{k-1}$. Indeed, for any $a, bin [0, ldots, N]$ we have $aoplus b in [0, ldots , N]$ . Since $|Soplus S| ge |S|$ and $Scap (Soplus S) = varnothing$ we get $2|S| le (N+1)$ or $|S| le 2^{k-1}$.



For the general case assume that $2^k le N < 2^{k+1} - 1$. Then on the one hand $|S| le 2^k$ since $Ssubset [0, ldots , 2^{k+1}-1]$ and on the other hand $|S| le 2^{k-1} + (N - 2^k) = N - 2^{k-1}$ since $|Scap [0, ldots , 2^k - 1]| le 2^{k-1}$. Therefore $3|S| le 2N$ or $|S| le frac{2N}{3}$.



Here is an example (found partly via computer search) which shows that density $frac{2}{3}$ is possible: put $N = 6*2^k$ and let $S$ be a set of all numbers consisting of $k + 3$ digits such that first three of them is from the following set $M = {(0, 1, 0), (0, 1, 1), (1, 0, 0), (1, 0, 1)}$. Note that $Soplus S cap S = varnothing$, all elements of $S$ are not greater than $N$ and $|S| = 4*2^k$. Therefore $frac{|S|}{N} = frac{2}{3}$.






share|cite|improve this answer











$endgroup$









  • 2




    $begingroup$
    The example can be described more simply as the integers whose fist two digits are $(0,1)$ or $(1,0)$, that is, $S = [2^{k+1}, 3 cdot 2^{k+1})$. Those digits sum to $1$, so in any element of $S oplus S$ they sum to $0$, whence $S$ is disjoint from $S oplus S$.
    $endgroup$
    – Noam D. Elkies
    47 mins ago














4












4








4





$begingroup$

Let $ain S$ be some fixed element. Note that $aoplus b le a + b$. Let $N$ be some big number. Put $M = [1, ldots , N]cap S$. We have $aoplus M cap M = varnothing$. We also have $aoplus M subset [1, ldots , N + a]$. Therefore $2|M| le N + a$ or $|M| le frac{N}{2} + frac{a}{2}$. Since $a$ is fixed taking limit $Nto infty$ yields the desired result.



UPD Here is an answer for a perhaps more interesting question: what is limsup of the biggest density of the subset of $[0, ldots , N]$ which is XOR-free. Note that if $N = 2^k - 1$ for some $kin mathbb{N}$ then $|S| le 2^{k-1}$. Indeed, for any $a, bin [0, ldots, N]$ we have $aoplus b in [0, ldots , N]$ . Since $|Soplus S| ge |S|$ and $Scap (Soplus S) = varnothing$ we get $2|S| le (N+1)$ or $|S| le 2^{k-1}$.



For the general case assume that $2^k le N < 2^{k+1} - 1$. Then on the one hand $|S| le 2^k$ since $Ssubset [0, ldots , 2^{k+1}-1]$ and on the other hand $|S| le 2^{k-1} + (N - 2^k) = N - 2^{k-1}$ since $|Scap [0, ldots , 2^k - 1]| le 2^{k-1}$. Therefore $3|S| le 2N$ or $|S| le frac{2N}{3}$.



Here is an example (found partly via computer search) which shows that density $frac{2}{3}$ is possible: put $N = 6*2^k$ and let $S$ be a set of all numbers consisting of $k + 3$ digits such that first three of them is from the following set $M = {(0, 1, 0), (0, 1, 1), (1, 0, 0), (1, 0, 1)}$. Note that $Soplus S cap S = varnothing$, all elements of $S$ are not greater than $N$ and $|S| = 4*2^k$. Therefore $frac{|S|}{N} = frac{2}{3}$.






share|cite|improve this answer











$endgroup$



Let $ain S$ be some fixed element. Note that $aoplus b le a + b$. Let $N$ be some big number. Put $M = [1, ldots , N]cap S$. We have $aoplus M cap M = varnothing$. We also have $aoplus M subset [1, ldots , N + a]$. Therefore $2|M| le N + a$ or $|M| le frac{N}{2} + frac{a}{2}$. Since $a$ is fixed taking limit $Nto infty$ yields the desired result.



UPD Here is an answer for a perhaps more interesting question: what is limsup of the biggest density of the subset of $[0, ldots , N]$ which is XOR-free. Note that if $N = 2^k - 1$ for some $kin mathbb{N}$ then $|S| le 2^{k-1}$. Indeed, for any $a, bin [0, ldots, N]$ we have $aoplus b in [0, ldots , N]$ . Since $|Soplus S| ge |S|$ and $Scap (Soplus S) = varnothing$ we get $2|S| le (N+1)$ or $|S| le 2^{k-1}$.



For the general case assume that $2^k le N < 2^{k+1} - 1$. Then on the one hand $|S| le 2^k$ since $Ssubset [0, ldots , 2^{k+1}-1]$ and on the other hand $|S| le 2^{k-1} + (N - 2^k) = N - 2^{k-1}$ since $|Scap [0, ldots , 2^k - 1]| le 2^{k-1}$. Therefore $3|S| le 2N$ or $|S| le frac{2N}{3}$.



Here is an example (found partly via computer search) which shows that density $frac{2}{3}$ is possible: put $N = 6*2^k$ and let $S$ be a set of all numbers consisting of $k + 3$ digits such that first three of them is from the following set $M = {(0, 1, 0), (0, 1, 1), (1, 0, 0), (1, 0, 1)}$. Note that $Soplus S cap S = varnothing$, all elements of $S$ are not greater than $N$ and $|S| = 4*2^k$. Therefore $frac{|S|}{N} = frac{2}{3}$.







share|cite|improve this answer














share|cite|improve this answer



share|cite|improve this answer








edited 1 hour ago

























answered 1 hour ago









Aleksei KulikovAleksei Kulikov

1,2061310




1,2061310








  • 2




    $begingroup$
    The example can be described more simply as the integers whose fist two digits are $(0,1)$ or $(1,0)$, that is, $S = [2^{k+1}, 3 cdot 2^{k+1})$. Those digits sum to $1$, so in any element of $S oplus S$ they sum to $0$, whence $S$ is disjoint from $S oplus S$.
    $endgroup$
    – Noam D. Elkies
    47 mins ago














  • 2




    $begingroup$
    The example can be described more simply as the integers whose fist two digits are $(0,1)$ or $(1,0)$, that is, $S = [2^{k+1}, 3 cdot 2^{k+1})$. Those digits sum to $1$, so in any element of $S oplus S$ they sum to $0$, whence $S$ is disjoint from $S oplus S$.
    $endgroup$
    – Noam D. Elkies
    47 mins ago








2




2




$begingroup$
The example can be described more simply as the integers whose fist two digits are $(0,1)$ or $(1,0)$, that is, $S = [2^{k+1}, 3 cdot 2^{k+1})$. Those digits sum to $1$, so in any element of $S oplus S$ they sum to $0$, whence $S$ is disjoint from $S oplus S$.
$endgroup$
– Noam D. Elkies
47 mins ago




$begingroup$
The example can be described more simply as the integers whose fist two digits are $(0,1)$ or $(1,0)$, that is, $S = [2^{k+1}, 3 cdot 2^{k+1})$. Those digits sum to $1$, so in any element of $S oplus S$ they sum to $0$, whence $S$ is disjoint from $S oplus S$.
$endgroup$
– Noam D. Elkies
47 mins ago


















draft saved

draft discarded




















































Thanks for contributing an answer to MathOverflow!


  • Please be sure to answer the question. Provide details and share your research!

But avoid



  • Asking for help, clarification, or responding to other answers.

  • Making statements based on opinion; back them up with references or personal experience.


Use MathJax to format equations. MathJax reference.


To learn more, see our tips on writing great answers.




draft saved


draft discarded














StackExchange.ready(
function () {
StackExchange.openid.initPostLogin('.new-post-login', 'https%3a%2f%2fmathoverflow.net%2fquestions%2f324044%2fxor-free-sets-maximum-density%23new-answer', 'question_page');
}
);

Post as a guest















Required, but never shown





















































Required, but never shown














Required, but never shown












Required, but never shown







Required, but never shown

































Required, but never shown














Required, but never shown












Required, but never shown







Required, but never shown







Popular posts from this blog

SQL Server 17 - Attemping to backup to remote NAS but Access is denied

Always On Availability groups resolving state after failover - Remote harden of transaction...

Restoring from pg_dump with foreign key constraints